Sei sulla pagina 1di 4

APMO 2001

1. For any positive integer n let S(n) be the sum of the digits in the decimal representation of n. Any positive integer obtained by removing several (at least one) digits
from the right-hand end of the decimal representation n is called a stump of n. Let T (n)
be the sum of all stumps of n. prove that n = S(n) + 9T (n).
soln 1. Let a be the unit digit of n and m be obtained by dropping the unit digit of
n. Then
n S(n) = 10m + a S(m) a
= 10m S(m)

Thus the proof cam be completed by using induction on the number of digits of n.
2. Find the largest positive integer N so that the number of integers in the set
{1, 2, . . . , N } which are divisible by 3 is equal to the number of integers which are divisible
by 5 or 7 (or both).
soln 2. This is equivalent to finding the largest positive integer solution of the equation

N
3

N
=
5

N
+
7

.
35

Let N = 35k + r be a solution. Then equation becomes

Now

35k+r2
3

35k + r
3

35k+r r
, i
3

r
i

j k
3
r
= 11k +
+
.
5
7

for i = 5, 7. So

35k + r 2
r
12r
r
11k + + =
,
3
5 7
35

which implies 70k r + 70 < 35 + 70 = 105. Then k 1 or equivalently N 69. Checking


values of N 69 shows that N = 65 is the answer.
3. Let two equal regular n-gons S and T be located in the plane such that their
intersection is a 2n-gon (n 3). The sides of the polygon S are coloured in red and the
sides of T in blue.
Prove that the sum of the lengths of the blue sides of the polygon S T is equal to
the sum of the lengths of the red sides.
soln 3. First observe that S T has 2n sides only if the sides of S T alternate.
Label the vertices of the red n-gon R1 , R2 , . . . Rn and the vertices of the blue n-gon
1

B1 , B2 , . . . , Bn . Place the n-gons so that the vertices are in the following clockwise order: B1 , R1 , B2 , R2 , . . . , Bn , Rn . Each of these vertices together with the opposite side
determines a triangle and all these triangles are similar. For each i = 1, . . . , n, we let
the lengths of the sides of the triangle determined by Bi be bi , ci , di in the clockwise order where bi is the side opposite Bi such that bi /b1 = ci /c1 = di /d1 = pi . We also
let the lengths of the sides of the triangle determined by Ri be ri , si , ti in the counter
clockwise order such that ri /b1 = si /c1 = ti /d1 = qi . Then we want to prove that
b1 + + bn = r1 + + rn or b1 (p1 + + pn ) = b1 (q1 + + qn ), where p1 = 1, or
equivalently,
Pn p = q where p = (p1 + + pn ), q = (q1 + + qn ). The perimeter of the blue
n-gon
Pn is i=1 (ci + di + ri ) = pc1 + pd1 + qb1 . Likewise the perimeter of the red n-gon is
i=1 (bi +si +ti ) = pb1 +qc1 +qd1 . Equating the two we have p(c1 +d1 b1 ) = q(c1 +d1 b1 ).
Since c1 + d1 b1 > 0 by the triangle inequality, we have p = q as required.
B1

...........
....... .........
.....
.......
..... 1
.......
.
.
.
.
.
.....
1............
.....
.....
.
.
.
.
..
.
....
1
.
.
.
.
.
....... ....... .............. ....... ....... ....... ....... ....... ....... ............. ....... ....... ....... ....... ....... ....
.....
.
.
.
...
.
.
.....
..
.
.
.
.
.
..
.
.....
......
.....
.....
1
......
.
.
.
.
.
1 ........
....
..... ... . 1
..... ....
.....
.
.
.
.
..... .........
.....
..
.....
.....
..... 2
.
.
.....
2.....
.....
.....
.
.
.
.
..
.
.
.
.....
..............
.
.
.
.
.
.
.
.
.
.
.
.
.
.
.
.
.
.
.
.
.
.
.
.
.
.
.
.
.
.
.
.
2 ......................................................
.......
.. ..
.
.
.
.
.
.
.
.
.
.
.
.
.
.
.
.
.
.
.
.
.
.
.
.
.
.
.
.
.
.
.
.
...
.....
..................................
......
2
..
.....
.......
2
.
.
.....

s2

R1

B2

R2

4. A point in the plane with a Cartesian system is called a mixed point if one of its
coordinates is rational and the other is irrational. Find all polynomials with real coefficients
such that their graphs do not contain any mixed point.
soln 4. Answer. All the polynomials of degree 1 with rational coefficients.
First Solution.It is well known that if f (r) Q for every r Q then all the
coefficients of f (x) are rational. In fact, for distinct rational numbers r0 , r1 , . . . , rn , where
n = deg f (x), we have the formula of Lagrange
f (x) =

n
X

f (ri )

i=0

Y x rj
,
ri rj
j6=i

where all the coefficients on the right are rational.


It is easy to see that all the polynomials of first degree with rational coefficients do
not contain mixed points while all polynomials of degree 0 do.
Suppose that f (x) = a0 +a1 x+. . .+an xn is a polynomial with ai Q and degree n 2.
We may assume that the coefficients of f (x) are integers, because the sets of solutions of
2

equations f (x) = r and af (x) = ar, where a is an integer, coincide. Moreover let us denote
g(x) = an1
f ( axn ). g(x) is a polynomial with integer coefficients whose leading coefficient
n
r has an
is 1. The equation f (x) = r has an irrational root if and only if g(x) = an1
n
irrational root and if and only if the equation f (x) f (0) = r f (0) has an irrational root.
Therefore, we may assume WLOG that f (x) has integer coefficients, an = 1 and a0 = 0.
Let r be a sufficiently large prime, such that
r > max{f (1), x1 , x2 , . . . , xk },
where {x1 , x2 , . . . , xk } denote the set of all real roots of f (x)x = 0. Thus f (1) < r < f (r).
It follows from the intermediate value theorem that there is at least s (1, r) such that
f (s) r = 0. If s Q, write s = p/q where p, q are coprime integers, and then substitute
into the equation f (s) = r, it is easy to see that q = 1, i.e., s is an integer. In the equation
f (s) = r, the left hand side is a multiple of s. Thus r | s which is impossible since s < r.
Thus s 6 Q and we have a mixed point.
Another soln:
(The main idea is that the graph of f (x) is very steep for large x and since the inverse
image of integers must be integers or irrational if f (x) has integer coefficients with leading
coefficient 1 well get a mixed point.)
Let f (x) be a polynomial of degree n(> 1) with rational coefficients. As before we
assume that the coefficients are integers, and the leading coefficient is 1. We have shown
before that if f (x) = r, where r is an integer, has a rational root, then the root must also
be an integer. Thus if f (a) = r, r Z, then either a Z or a 6 Q.
Now f (x + 1) f (x) = nxn1 + p(x) where p(x) is a polynomial of degree n 2. Let
k be an integer strictly larger than the maximum of the absolute values of the coefficients
of p(x). We also require that k 2. When x = k,
f (x + 1) f (x) nxn1 (n 1)kxn2 k n1 > 2.
Thus there are at least three integers in [f (k), f (k+1)]. For each integer c [f (k), f (k +1)]
the equation f (x) = c has a solution in [k, k + 1] which is either an integer or an irrational
number . Since there are only two integers in the interval, there must be a mixed point.
5. Find the greatest integer n, such that there are n + 4 points A, B, C, D, X1 , . . . , Xn
in the plane with AB 6= CD that satisfy the following condition: for each i = 1, 2, . . . , n,
triangles ABXi and CDXi are equal.
soln Let AB = p and CD = q with p > q. If ABX equals CDX, then either AX = r
or BX = r. Assume with loss of generality that AX = r. Similarly, assume that CX = p.
Let a (q) be the circle with radius q and centre A. The other circles b (q), c (p), d (p)
3

are similarly defined. Then X lies on the intersection of a with c . Thus n 8 as there
are at most 8 points that have the given properties: the 8 intersection points of the circles
x , y , x = a, b, y = c, d.
Suppose that X, Y are the intersections of a with c and they both have the given
properties. Then we must have BX = DX and BY = DY . Thus the line XY is the
perpendicular bisector of the segment BD. Since XY is also perpendicular to AC, we see
that AC||BD.
Let W be a point of intersection of b with d . If ABW is equal to CDW , then
AW = BW , i.e. W is on the perpendicular bisector of AB. But that means q = BW >
W D = p, a contradiction. Therefore ABW is not equal to CDW . Thus the two pairs
of circles (a , c ) and (b , d ) can contribute at most two points that satisfy the given
conditions. Thus n 4.
B.
....
......
... ..
.. ....
.
.
.
.........
............ ....................... ......
.......
.. ...... ....
.....
... .............
.....
.
.
.
.
.
......
...
...
...... ...
...
...
.... ..
...
...
..... .....
.
.
.
....
.
...
.... ..
..
....
..... ...
........
...
....... ....
.... ........
.
.
.
...
.
.
.....
.....
...
.. .. ...
.....
...
.....
. .. .. ...
... ........ ................ ......... .... ........
........................
..... .....................
.................................................................................
.. . .
....... ........
..... ........................................................ ... ..........
.
.....
.....
...
.
.
.
.
... .... ....
.....
.
.
.
.
.
.
.
.
...
...
. ..
..
...
...
... ..... ....
...
.
.
.
.
...
...
.
......
.
..
.
.
...
...
.
.
.
..
.
.
.
...
.
.
...
......
.
.
.
.
...
.
...
.
......
.
.
.
.
...
.
...
.
.
.
.
.
.
.
....
...
... ..
.....
.
...
... ..
...
.....
.
..
.....
.....
...
.....
...
......
.....
...
...
.
.
.
.
.
.
.
.
.
....
...
.....
.
.
.
.
.
.....
...
.
..... ....
...
...
..... ..
...
...
..... ...
...
...
.
.
.
.
.
.
...
..
...
...
...
...
.....
.....
.
.....
.
.
.
......
...
.....
......
.......
......
..........
.......
...........................................

p p
C

Since ABP is equal to CDP , BAP = P CD.


The following configuration of a regular hexagon gives an example with n = 4.
A

.......
.............. ..............
............................................................
........
.........
.........
.
... ....
.
.
. .
... ...
... .....
... ...
.
.
. ...
... ...
.
.
... ..
.........
......
.....
..............................................................................................
.
.
.
.
.
.......
........
.
.
. .. ..
..... ...
... ..... .....
... .... .....
.
.
.
.
... ..
.
...
... ...
... ...
...
...
... ...
... ...
...
...
..... ...
..........
...
...
.
.
.
.
.
.
.
.
.
.
.
.. .
.
...................................................................................................................................................................
............
.
.
.
....................

X1

X2

X3

X4

Potrebbero piacerti anche